[Date Prev][Date Next][Thread Prev][Thread Next][Date Index][Thread Index]

[obm-l] Re: [obm-l] Re: [obm-l] Análise Real



Manuel (e todos os integrantes desta lista)
Bom dia. 

-- Mensagem original --

>Bernardo,
>
>  Boa tarde,
>
>  Só dois comentários:
>
>  (1) Há algo "estranho" com o "corolário", ele é completamente trivial,
>mas não sei como concluir do exercício original esse resultado. 

Você tem toda a razão... na hora de escrever, confundi-me completamente
e saiu este absurdo... era, realmente, Q != I(A_i), com A_i abertos, como
você corretamente corrigiu.

Veja o
>seguinte, Q não pode ser a renuião enumerável de abertos, simplesmente
>porque cada aberto não vazio de R contém um inervalo aberto (a,b) não
>vazio. Logo se Q fosse uma reunião de abertos (enumerável ou não) Q
>conteria (a,b). Iso é absurdo pois R-Q é denso em R. Talvez o corolário
>seja "Q não é a intercecção enumerável de abertos". De fato isso segue-se
>imediatamente do exercício proposto, por passagem ao complementar.
>
>  (2) Não sei exatamente o contexto em que o exercício apareceu, às vezes
>quando se fala em R, esconde-se quando se está usando Baire. Você precisa
>saber alguma coisa, por exemplo que R não pode ser escrito como reunião
>enumerável de fechados sem interior [pode chamar isso propriedade de Baire
>da reta] ou algo equivalente para fazer o exercício (o que foi usado na
>demonstração do outro email foi algo equivalente). Se você souber dessa
>propriedade que enunciei acima, uma demonstração "alternativa" (que, no
>fundo é exatamente igual) é a seguinte.

Isso começa a fazer sentido... pois o exercício anterior pedia para provar
a propriedade de em R, U(F_i), F_i fechados com interior vazio, ser ainda
de interior vazio.

>
>    Suponha, por absurdo, que existem subconjuntos fechados de R, F_1,
>F_2,..., F_n,... tais que a reunião de todos os F_n seja R-Q.
>
>    Como Q não tem interior (pois nenhum intervalo aberto da reta, não
>vazio, está contido em Q) segue-se que cada F_n tem interior vazio.

Esta parte eu não entendi... Eu achei que seria suficiente para F_n terem
interior vazio o fato de, caso contrário, conterem algum intervalo da forma
(a, b) e portanto F_n possuiria algum racional (pois todo intervalo aberto
contém racionais). Mas aí a interseção não poderia ser R-Q.

>
>   Qomo Q é enumerável tome {q_k, k em N} uma enumeração de Q e defina
>T_j={q_j}, j=1,2,...
>
>  Claro que cada T_j é fechado e de interior vazio.
>
>  Então R = (R-Q) U Q seria a reunião dos F_n com os T_j. Então ter-se-ia
>escrito R como uma reunião enumerável de fechados sem interior, o que
>contraria a aupramencionada propriedade de Baire da reta.
>
>Manuel Garcia

Muito obrigado, mesmo.
Até mais
Bernardo



------------------------------------------
Use o melhor sistema de busca da Internet
Radar UOL - http://www.radaruol.com.br



=========================================================================
Instruções para entrar na lista, sair da lista e usar a lista em
http://www.mat.puc-rio.br/~nicolau/olimp/obm-l.html
=========================================================================